Last visit was: 15 Jun 2025, 10:28 It is currently 15 Jun 2025, 10:28
Close
GMAT Club Daily Prep
Thank you for using the timer - this advanced tool can estimate your performance and suggest more practice questions. We have subscribed you to Daily Prep Questions via email.

Customized
for You

we will pick new questions that match your level based on your Timer History

Track
Your Progress

every week, we’ll send you an estimated GMAT score based on your performance

Practice
Pays

we will pick new questions that match your level based on your Timer History
Not interested in getting valuable practice questions and articles delivered to your email? No problem, unsubscribe here.
Close
Request Expert Reply
Confirm Cancel
555-605 Level|   Weaken|                              
User avatar
jabhatta2
Joined: 15 Dec 2016
Last visit: 21 Apr 2023
Posts: 1,304
Own Kudos:
Given Kudos: 188
Posts: 1,304
Kudos: 273
Kudos
Add Kudos
Bookmarks
Bookmark this Post
avatar
Justinseaman
Joined: 11 Nov 2020
Last visit: 18 Dec 2020
Posts: 1
Given Kudos: 5
Posts: 1
Kudos: 0
Kudos
Add Kudos
Bookmarks
Bookmark this Post
User avatar
KarishmaB
Joined: 16 Oct 2010
Last visit: 15 Jun 2025
Posts: 16,046
Own Kudos:
73,680
 [2]
Given Kudos: 472
Location: Pune, India
Expert
Expert reply
Active GMAT Club Expert! Tag them with @ followed by their username for a faster response.
Posts: 16,046
Kudos: 73,680
 [2]
2
Kudos
Add Kudos
Bookmarks
Bookmark this Post
User avatar
GMATNinja
User avatar
GMAT Club Verbal Expert
Joined: 13 Aug 2009
Last visit: 15 Jun 2025
Posts: 7,325
Own Kudos:
68,236
 [1]
Given Kudos: 1,943
Status: GMAT/GRE/LSAT tutors
Location: United States (CO)
GMAT 1: 780 Q51 V46
GMAT 2: 800 Q51 V51
GRE 1: Q170 V170
GRE 2: Q170 V170
Products:
Expert
Expert reply
GMAT 2: 800 Q51 V51
GRE 1: Q170 V170
GRE 2: Q170 V170
Posts: 7,325
Kudos: 68,236
 [1]
1
Kudos
Add Kudos
Bookmarks
Bookmark this Post
Justinseaman
There is no evidence to suggest that the inbound population will be any younger than the current population. What if a new retirement village opened up on the island? B or D feel equally specious as potential answers here.
Quote:
(D) The overall population of Canatria is growing steadily.
(D) does not require that the inbound population be younger than the current population. Even if the increase in Canatria’s population is the result of new retirement homes, older people will still eat at fast-food restaurants some. Because the overall pool of potential customers for Canatrian restaurants will be larger, they will be able to afford a smaller percentage of potential customers converting into actual customers.

Also, (D) makes it possible that the number of young customers remains constant and the number of old customers decreases. While this would lead to the average age of the Canatrian population rising, it would actually INCREASE the overall number of the restaurants’ customers. So (D) still weakens the argument that the number of fast-food restaurants is likely to decrease.

I hope that helps!
User avatar
CEdward
Joined: 11 Aug 2020
Last visit: 14 Apr 2022
Posts: 1,214
Own Kudos:
Given Kudos: 332
Posts: 1,214
Kudos: 244
Kudos
Add Kudos
Bookmarks
Bookmark this Post
Why isn't A correct? What it suggests is that there is increased demand so it’s not necessarily the case that the number of restaurants will decrease.. After all, why would restaurants get bigger if fewer customers are coming in?

GMATNinja
User avatar
GMATNinja
User avatar
GMAT Club Verbal Expert
Joined: 13 Aug 2009
Last visit: 15 Jun 2025
Posts: 7,325
Own Kudos:
68,236
 [3]
Given Kudos: 1,943
Status: GMAT/GRE/LSAT tutors
Location: United States (CO)
GMAT 1: 780 Q51 V46
GMAT 2: 800 Q51 V51
GRE 1: Q170 V170
GRE 2: Q170 V170
Products:
Expert
Expert reply
GMAT 2: 800 Q51 V51
GRE 1: Q170 V170
GRE 2: Q170 V170
Posts: 7,325
Kudos: 68,236
 [3]
3
Kudos
Add Kudos
Bookmarks
Bookmark this Post
CEdward
Why isn't A correct? What it suggests is that there is increased demand so it’s not necessarily the case that the number of restaurants will decrease.. After all, why would restaurants get bigger if fewer customers are coming in?

GMATNinja
Quote:
(A) Fast-food restaurants in Canatria are getting bigger, so each one can serve more customers.
The problem with (A) is that we don’t actually know why fast-food restaurants want to serve more customers. It’s possible that, as you said, the restaurants are responding to an increase in demand. In that case, perhaps (A) would weaken the argument.

But an increase in restaurant capacity is not necessarily indicative of an increase in overall customer demand. It could be that the restaurants are simply increasing their capacity in anticipation of other fast-food restaurants closing. They may expect to serve more customers at their particular restaurant, even though overall demand is likely to decrease. If chains are serving more customers per restaurant and demand remains steady or decreases, then many restaurants will likely be forced to close.

All of this also is based on the assumption that the restaurants are right in their expectations. We can assume that (A) is true. But we can’t pretend to know why the restaurants do what they do, and we also can’t necessarily assume that the restaurants are right in their thinking. So, we really can’t conclude that (A) strengthens or weakens the argument. Eliminate (A).

I hope that helps!
User avatar
M838TE
Joined: 25 Jul 2020
Last visit: 30 Jul 2023
Posts: 46
Own Kudos:
Given Kudos: 15
Posts: 46
Kudos: 6
Kudos
Add Kudos
Bookmarks
Bookmark this Post
GMATNinja
scarlatti
Vyshak
Younger people eat in fastfood. Average age of current population is increasing - Number of fastfood restaurants will decrease

Possible weakener: As the current population ages, younger people are added to the population

A) Fast-food restaurants in Canatria are getting bigger, so each one can serve more customers. - Incorrect - Out of context. We are not bothered about the increase in capacity of fast food restaurants.

B) Some older people eat at fast-food restaurants more frequently than the average young person. - Incorrect - Irrelevant

C) Many people who rarely eat in fast-food restaurants nevertheless eat regularly in restaurants. - Incorrect - Strengthens

D) The overall population of Canatria is growing steadily. - Correct

E) As the population of Canatria gets older, more people are eating at home. - Incorrect - Supports the argument

Answer: D

I have a problem with D. Just because Canatria´s population increased, it doesn´t mean that the number of young people in Canatria will increase, since that same increase in the population could have been due to immigration for example, and we can´t say that immigrants are young.
Vyshak, it's true that choice (D) might not weaken the argument. We can think of scenarios, like the one you described, in which choice (D) would have no effect. But remember that we are looking for an answer choice that most seriously weakens the argument, not an answer choice that definitely weakens the argument.

The author says that the number of fast-food restaurants is likely to decrease because the average age is increasing. Thus, according to the author, the number of young people will probably decrease. Choice (D) seriously undermines this reasoning. If the overall population is growing steadily ("steadily" is a key word here), then that will probably counteract the effect of the aging.

Sure, we might have a steady influx of older immigrants, but if we are talking about steady growth of the overall population, it is more likely that we will see increases across all ages. Choice (D) does not disprove the author's logic with 100% certainty, but it does seriously weaken the argument.

I hope that helps!

I am still a little hazy on this one GMATNinja. I am number guy, I get that if overall population increase, the whole demographic could grow proportionally. Thus, it could offset the number of old people intown and continue to have a steady number of fast food customer base. However, the "average" continues to grow as the problem stated. I thought this would overwrite any assumption *we thought* the author were making. In a way, I think the statement is trying to say there are way more old people and will continue to outweigh the young.
I probably do not make any sense here please correct me if I am wrong or assuming things. I am doing this assumption identification practice, and the one made in this excerpt: 1) no other possible factors would suddenly increase the demand on fast food 2) the overall age of the population will continue to grow old regardless of possible outside factors like migrations. I was ohonestly only looking at choices that resembles 1).
User avatar
GMATNinja
User avatar
GMAT Club Verbal Expert
Joined: 13 Aug 2009
Last visit: 15 Jun 2025
Posts: 7,325
Own Kudos:
68,236
 [4]
Given Kudos: 1,943
Status: GMAT/GRE/LSAT tutors
Location: United States (CO)
GMAT 1: 780 Q51 V46
GMAT 2: 800 Q51 V51
GRE 1: Q170 V170
GRE 2: Q170 V170
Products:
Expert
Expert reply
GMAT 2: 800 Q51 V51
GRE 1: Q170 V170
GRE 2: Q170 V170
Posts: 7,325
Kudos: 68,236
 [4]
4
Kudos
Add Kudos
Bookmarks
Bookmark this Post
M838TE
GMATNinja
scarlatti
Younger people eat in fastfood. Average age of current population is increasing - Number of fastfood restaurants will decrease

Possible weakener: As the current population ages, younger people are added to the population

A) Fast-food restaurants in Canatria are getting bigger, so each one can serve more customers. - Incorrect - Out of context. We are not bothered about the increase in capacity of fast food restaurants.

B) Some older people eat at fast-food restaurants more frequently than the average young person. - Incorrect - Irrelevant

C) Many people who rarely eat in fast-food restaurants nevertheless eat regularly in restaurants. - Incorrect - Strengthens

D) The overall population of Canatria is growing steadily. - Correct

E) As the population of Canatria gets older, more people are eating at home. - Incorrect - Supports the argument

Answer: D

I have a problem with D. Just because Canatria´s population increased, it doesn´t mean that the number of young people in Canatria will increase, since that same increase in the population could have been due to immigration for example, and we can´t say that immigrants are young.
Vyshak, it's true that choice (D) might not weaken the argument. We can think of scenarios, like the one you described, in which choice (D) would have no effect. But remember that we are looking for an answer choice that most seriously weakens the argument, not an answer choice that definitely weakens the argument.

The author says that the number of fast-food restaurants is likely to decrease because the average age is increasing. Thus, according to the author, the number of young people will probably decrease. Choice (D) seriously undermines this reasoning. If the overall population is growing steadily ("steadily" is a key word here), then that will probably counteract the effect of the aging.

Sure, we might have a steady influx of older immigrants, but if we are talking about steady growth of the overall population, it is more likely that we will see increases across all ages. Choice (D) does not disprove the author's logic with 100% certainty, but it does seriously weaken the argument.

I hope that helps!

I am still a little hazy on this one GMATNinja. I am number guy, I get that if overall population increase, the whole demographic could grow proportionally. Thus, it could offset the number of old people intown and continue to have a steady number of fast food customer base. However, the "average" continues to grow as the problem stated. I thought this would overwrite any assumption *we thought* the author were making. In a way, I think the statement is trying to say there are way more old people and will continue to outweigh the young.
I probably do not make any sense here please correct me if I am wrong or assuming things. I am doing this assumption identification practice, and the one made in this excerpt: 1) no other possible factors would suddenly increase the demand on fast food 2) the overall age of the population will continue to grow old regardless of possible outside factors like migrations. I was ohonestly only looking at choices that resembles 1).
The average age and number of young people in Canatria can both increase.

For example, let’s say that Canatria currently has 200 young people and 400 old people. In ten years, it may have 300 young people and 800 old people. Old people would now make up a greater proportion of the population, and the average age of the town would increase.

But because the overall population has grown, there is actually a larger number of young people in the town and thus a larger base of customers for fast-food restaurants to attract. Therefore, it is unlikely that the number of fast-food restaurants will decrease, and (D) weakens the argument.

I hope that helps!
avatar
LetsGetStartedNow
Joined: 01 May 2021
Last visit: 12 May 2021
Posts: 2
Posts: 2
Kudos: 0
Kudos
Add Kudos
Bookmarks
Bookmark this Post
GMATNinja
Quote:
Dear Experts, please assist me here: Isn't Option B valid? Although it states that "Some" older people eat more frequently than the younger people, the statement still attacks the conclusion made by the author! Although option B says "Some", the option is still strong enough to arrest the decrease of the fast food restaurants
Quote:
B) Some older people eat at fast-food restaurants more frequently than the average young person.
Even if some older people eat at fast-food restaurants more frequently than the average young person, it wouldn't change the evidence on which the argument is based: "Customers at these restaurants tend to be young; in fact, studies have shown that the older people get, the less likely they are to eat in fast-food restaurants."

This evidence only suggests that older people are less likely to eat in fast-food restaurants and does not suggest that ALL older people eat at fast-food restaurants less frequently than the average young person. Statement (B) does not interfere with the author's argument and can be eliminated.

I hope that helps!

Hi Charles,
I'm still struggling with this one. I rejected "D" because
a.) it was questioning a premise that the average age of the town is increasing
b.) population increase can be from new birth or migration. You've explained the this one, and I get that.
I picked "A" as my answer because I thought of this stimulus as Cause (increase in average age) > Effect (decrease in number of restaurants) and "A" provides an alternate cause for the number of restaurant to come down.
Let me know; thanks!
User avatar
GMATNinja
User avatar
GMAT Club Verbal Expert
Joined: 13 Aug 2009
Last visit: 15 Jun 2025
Posts: 7,325
Own Kudos:
68,236
 [2]
Given Kudos: 1,943
Status: GMAT/GRE/LSAT tutors
Location: United States (CO)
GMAT 1: 780 Q51 V46
GMAT 2: 800 Q51 V51
GRE 1: Q170 V170
GRE 2: Q170 V170
Products:
Expert
Expert reply
GMAT 2: 800 Q51 V51
GRE 1: Q170 V170
GRE 2: Q170 V170
Posts: 7,325
Kudos: 68,236
 [2]
2
Kudos
Add Kudos
Bookmarks
Bookmark this Post
LetsGetStartedNow
GMATNinja
Quote:
Dear Experts, please assist me here: Isn't Option B valid? Although it states that "Some" older people eat more frequently than the younger people, the statement still attacks the conclusion made by the author! Although option B says "Some", the option is still strong enough to arrest the decrease of the fast food restaurants
Quote:
B) Some older people eat at fast-food restaurants more frequently than the average young person.
Even if some older people eat at fast-food restaurants more frequently than the average young person, it wouldn't change the evidence on which the argument is based: "Customers at these restaurants tend to be young; in fact, studies have shown that the older people get, the less likely they are to eat in fast-food restaurants."

This evidence only suggests that older people are less likely to eat in fast-food restaurants and does not suggest that ALL older people eat at fast-food restaurants less frequently than the average young person. Statement (B) does not interfere with the author's argument and can be eliminated.

I hope that helps!

Hi Charles,
I'm still struggling with this one. I rejected "D" because
a.) it was questioning a premise that the average age of the town is increasing
b.) population increase can be from new birth or migration. You've explained the this one, and I get that.
I picked "A" as my answer because I thought of this stimulus as Cause (increase in average age) > Effect (decrease in number of restaurants) and "A" provides an alternate cause for the number of restaurant to come down.
Let me know; thanks!
The author's conclusion is that "the number of fast-food restaurants is likely to decrease." Our task is to weaken this conclusion.

(A), as you've pointed out, gives us another reason to believe that the number of these restaurants will decrease: if each one can serve more customers, then maybe Cantaria won't need so many fast-food restaurants.

Sure, this is a different reason then the one provided by the author -- but at the end of the day, it still strengthens the conclusion that the number of fast-food restaurants will decrease. So, we certainly can't say that it weakens the author's argument.

If the author had argued that the ONLY reason that the number of fast-food restaurants will decrease is because of the demographic information cited in the passage, then (A) WOULD weaken that argument. But the author doesn't specify that the aging population is the only explanation. He/she just argues that the number of restaurants will decrease. (A) provides another leg of support for this conclusion, so it doesn't weaken the argument.

I hope that helps!
User avatar
AnishPassi
Joined: 16 Jul 2014
Last visit: 15 Jun 2025
Posts: 106
Own Kudos:
569
 [5]
Given Kudos: 18
Status:GMAT Coach
Affiliations: The GMAT Co.
Concentration: Strategy
Schools: IIMA  (A)
GMAT 1: 760 Q50 V41
Expert
Expert reply
Schools: IIMA  (A)
GMAT 1: 760 Q50 V41
Posts: 106
Kudos: 569
 [5]
4
Kudos
Add Kudos
1
Bookmarks
Bookmark this Post
The Story

Fast-food restaurants make up 45 percent of all restaurants in Cantaria. – Say there are 100 restaurants in all of Cantaria, 45 of those would be Fast-Food restaurants.

Customers at these restaurants tend to be young; in fact, studies have shown that the older people get, the less likely they are to eat in fast-food restaurants. – A typical customer at these Fast-Food restaurants is young. According to some studies, with age people tend to eat less often in Fast-Food restaurants. (Both these ideas are about general trends. There very well could be old people who eat at such restaurants. And there very well could be young people who do not.)

Since the average age of the Canatrian population is gradually rising and will continue to do so, the number of fast-food restaurants is likely to decrease.– The average age of Cantaria’s population is gradually increasing. This trend will continue.
Therefore, the number of Fast-Food restaurants is likely to decrease.


Gist:
The number of Fast-Food restaurants in Cantaria is likely to decrease (main point).
Because:
1. Fast-food restaurants are frequented more by young people than they are by old people.
2. With age, people tend to eat less in fast-food restaurants.
3. The average age of Cantaria’s population will keep on rising.


Gap(s) in logic:
Notice the wording: The number of Fast-food restaurants is likely to decrease.
The argument only gives us relative figures.
[*]% of Fast-food restaurants
[*]Average age trends.
To conclude about the number of restaurants based on these general trends is flawed.

    1. If the average age of the population is going up, that would mean that young people would constitute a smaller percentage of the population. But, it doesn’t necessarily mean that the number of young people would also be reducing. If the overall population is increasing, Cantaria could still have more young people even if as a percentage, they now make up of a smaller part.
    2. Even if, let’s say, the target consumers of fast-food restaurants decrease (or their revenues or profits decrease), there could be a trend that fast-food restaurants reduce the capacity per restaurant and increase more smaller kiosks and stalls, thereby still increasing the number of fast-food restaurants.


Framework:
We’re looking for an answer choice that would reduce our confidence in the main point that the number of fast-food restaurants is likely to decrease.
i.e., when I add the correct answer choice to the argument, I get a sense that perhaps the number of fast-food restaurants will not reduce.


Answer choice analysis


A. Fast-food restaurants in Canatria are getting bigger, so each one can serve more customers.
Incorrect. This one is going in the opposite direction. If each fast-food restaurant can serve more customers, I believe more than I did earlier that the number of fast-food restaurants will decrease. If each restaurant can serve more customers they will probably need fewer such restaurants. This answer choice strengthens the argument.

B. Some older people eat at fast-food restaurants more frequently than the average young person.
Incorrect. The argument deals with likelihoods and trends.
* Customers tend to be young.
* The older people get the less likely they are to eat in a fast-food restaurant.

So, there very well may be some older people who eat at fast-food restaurants more frequently than the average young person.
This piece of information does not change my belief in the argument.

Note: This answer choice is not indicating any trend. Nothing like:
* the average age of fast-food restaurant customers is going up
* or that older people are starting to eat at fast-food restaurant more frequently than before.

Let’s take an example:
Say Americans tend to be taller than Indians.
Does this statement mean that EVERY American is taller than the average Indian?
No, right? Some Indians could be taller than the average American.
Similarly, the given argument does not indicate that every older person eats at fast-food restaurants less frequently than the average young person. This option does not change my belief in any of the trends mentioned in the passage.
No impact.

C.Many people who rarely eat in fast-food restaurants nevertheless eat regularly in restaurants.
Incorrect. Irrelevant. What people do when they do not eat in fast-food restaurants doesn’t matter. This answer choice doesn’t help me understand anything about whether the number of fast-food restaurants will reduce.
No impact.

D. The overall population of Canatria is growing steadily.
Correct. Bingo! The argument was based on the fact that the average age of the population is rising (i.e. the proportion of young people in the population is decreasing). Now that I learn that the overall population is growing steadily, I’m thinking that in that case perhaps the number of young people will not go down in Cantaria, even if their proportion is decreasing. This answer choice is in line with the first gap mentioned above.

Note: This answer choice does not guarantee that the number of young people will increase. But that’s the beauty of ‘weaken’ questions. We’re not looking for an answer choice that makes the conclusion impossible. We’re looking for an answer choice that reduces our confidence. This one does that.

E. As the population of Canatria gets older, more people are eating at home.
Incorrect. This could be the reason for why ‘the older people get, the less likely they are to eat in fast-food restaurants’. We already knew that older people are less likely to eat in fast-food restaurants. Now we learn that perhaps they eat more of their meals at home. Beyond that, this answer choice doesn’t add anything to the argument.
User avatar
rmahe11
Joined: 13 Oct 2023
Last visit: 09 Jun 2025
Posts: 115
Own Kudos:
Given Kudos: 99
Posts: 115
Kudos: 22
Kudos
Add Kudos
Bookmarks
Bookmark this Post
GMATNinja
Quote:
Dear Experts, please assist me here: Isn't Option B valid? Although it states that "Some" older people eat more frequently than the younger people, the statement still attacks the conclusion made by the author! Although option B says "Some", the option is still strong enough to arrest the decrease of the fast food restaurants
Quote:
B) Some older people eat at fast-food restaurants more frequently than the average young person.
Even if some older people eat at fast-food restaurants more frequently than the average young person, it wouldn't change the evidence on which the argument is based: "Customers at these restaurants tend to be young; in fact, studies have shown that the older people get, the less likely they are to eat in fast-food restaurants."

This evidence only suggests that older people are less likely to eat in fast-food restaurants and does not suggest that ALL older people eat at fast-food restaurants less frequently than the average young person. Statement (B) does not interfere with the author's argument and can be eliminated.

I hope that helps!
­
Ok , If the overall populations is growing wont it be true that people can be coming from different counties or cities etc . The option itself is a gap right ? I know B also doesnt specify how many older people eat at fast foods but "some" means at least one right ?. So how come B is incorrect 
User avatar
GMATNinja
User avatar
GMAT Club Verbal Expert
Joined: 13 Aug 2009
Last visit: 15 Jun 2025
Posts: 7,325
Own Kudos:
68,236
 [2]
Given Kudos: 1,943
Status: GMAT/GRE/LSAT tutors
Location: United States (CO)
GMAT 1: 780 Q51 V46
GMAT 2: 800 Q51 V51
GRE 1: Q170 V170
GRE 2: Q170 V170
Products:
Expert
Expert reply
GMAT 2: 800 Q51 V51
GRE 1: Q170 V170
GRE 2: Q170 V170
Posts: 7,325
Kudos: 68,236
 [2]
2
Kudos
Add Kudos
Bookmarks
Bookmark this Post
rmahe11

GMATNinja

Quote:
Dear Experts, please assist me here: Isn't Option B valid? Although it states that "Some" older people eat more frequently than the younger people, the statement still attacks the conclusion made by the author! Although option B says "Some", the option is still strong enough to arrest the decrease of the fast food restaurants

Quote:
B) Some older people eat at fast-food restaurants more frequently than the average young person.

Even if some older people eat at fast-food restaurants more frequently than the average young person, it wouldn't change the evidence on which the argument is based: "Customers at these restaurants tend to be young; in fact, studies have shown that the older people get, the less likely they are to eat in fast-food restaurants."

This evidence only suggests that older people are less likely to eat in fast-food restaurants and does not suggest that ALL older people eat at fast-food restaurants less frequently than the average young person. Statement (B) does not interfere with the author's argument and can be eliminated.

I hope that helps!

Ok , If the overall populations is growing wont it be true that people can be coming from different counties or cities etc . The option itself is a gap right ? I know B also doesnt specify how many older people eat at fast foods but "some" means at least one right ?. So how come B is incorrect

­Not sure what you're asking about (D), but see if this post helps: https://gmatclub.com/forum/fast-food-re ... l#p2751175

As for (B), the passage tells us that "customers at these restaurants TEND to be young; in fact, studies have shown that the older people get, the LESS LIKELY they are to eat in fast-food restaurants." In other words, the argument is only concerned with the GENERAL trends.

Sure, there might be some older folks who crush the Cantaria In & Out Burger on a regular basis (mmm... burgers), but that doesn't conflict with anything in the argument. The author would agree that SOME older folks might eat more fast food than the average younger person. That does not change the fact that the fast food customers TEND to be young.

So even if (B) is a very mild weakener, it's something that's already implied by the argument itself -- so (B) doesn't do anything new to weaken the argument at all.

I hope that helps!
   1   2 
Moderators:
GMAT Club Verbal Expert
7325 posts
GMAT Club Verbal Expert
235 posts